Đến nội dung

Hình ảnh

$\sqrt{\frac{2a}{a+b}}+\sqrt{\frac{2b}{b+c}}+\sqrt{\frac{2c}{c+a}}\leq 3$

* * * - - 1 Bình chọn

  • Please log in to reply
Chủ đề này có 2 trả lời

#1
khanghaxuan

khanghaxuan

    Trung úy

  • Thành viên
  • 969 Bài viết

Cho $a,b,c>0$ . CMR : 

$\sqrt{\frac{2a}{a+b}}+\sqrt{\frac{2b}{b+c}}+\sqrt{\frac{2c}{c+a}}\leq 3$


Điều tôi muốn biết trước tiên không phải là bạn đã thất bại ra sao mà là bạn đã chấp nhận nó như thế nào .

- A.Lincoln -

#2
canhhoang30011999

canhhoang30011999

    Thiếu úy

  • Thành viên
  • 634 Bài viết

Cho $a,b,c>0$ . CMR : 

$\sqrt{\frac{2a}{a+b}}+\sqrt{\frac{2b}{b+c}}+\sqrt{\frac{2c}{c+a}}\leq 3$

$\left (\sum \sqrt{\frac{2a}{a+b}} \right )^{2}$

=$\left ( \sum \sqrt{\frac{2a(a+c)}{(a+b)(a+c)}} \right )^{2}\leq 2(a+b+c) (\sum \frac{2a}{(a+b)(a+c)})$(BCS)

$= \frac{8(a+b+c)(ab+bc+ca)}{(a+b)(b+c)(c+a)}\leq 9$

vậy ta  có đpcm


Bài viết đã được chỉnh sửa nội dung bởi canhhoang30011999: 25-05-2015 - 15:30


#3
quanchun98

quanchun98

    Binh nhất

  • Thành viên
  • 33 Bài viết

Đổi biến $(x, y, z)=(\sqrt{\frac{b}{a}}, \sqrt{\frac{c}{b}},\sqrt{\frac{a}{c}})$ thì ta có xyz=1 và cần chứng minh $\sum \frac{1}{\sqrt{1+x^{2}}}\leq \frac{3}{\sqrt{2}}$. Không mất tính tổng quát, giả sử $z=max\left \{ x,y,z \right \}$ thì $z\geq 1, xy\leq 1$. Ta chứng minh bổ đề sau: $\frac{1}{1+x^{2}}+\frac{1}{1+y^{2}}\leq \frac{2}{1+xy}$ với $xy\leq 1$, cái này biến đổi tương đương là ra. Ta có $\sum \frac{1}{\sqrt{1+x^{2}}}\leq \sqrt{2(\frac{1}{1+x^{2}}+\frac{1}{1+y^{2}})}+\frac{1}{\sqrt{1+z^{2}}}\leq 2\sqrt{\frac{z}{1+z}}+\frac{\sqrt{z}}{1+z}=\frac{\sqrt{2}\sqrt{2z(z+1)}}{1+z}+\frac{\sqrt{z}}{1+z}\leq \frac{3z+1}{\sqrt{2}(z+1)}+\frac{\sqrt{2}}{1+z}=\frac{3}{\sqrt{2}}$ đpcm.


Bài viết đã được chỉnh sửa nội dung bởi quanchun98: 25-05-2015 - 15:52





1 người đang xem chủ đề

0 thành viên, 1 khách, 0 thành viên ẩn danh